====== Geometric Sequences and Sums 기하수열과 합 (그리고 합의 증명) ====== ====== Sequence ====== 숫자가 (대개는 숫자를 칭한다) 규칙을 가지고 연속적으로 배치되어 있는 집합을 말한다. $ 3, 5, 7, 9, . . . $ 는 2씩 증가하는 규칙을 가진 sequence이다. 이는 | 3 | 1st term | | 5 | 2nd term | | 7 | 3rd term | | 9 | 4th term | | . . . | . . . . | 뒤의 $. . .$ 는 이런 배열이 무한히 계속됨을 말한다. ====== Geometric Sequence ====== Geometric Sequence란 숫자의 sequence에서 뒤의 숫자가 앞의 숫자에 특정한 상수를 곱하여 구해지는 sequence를 말한다. 예를 들면 1, 2, 4, 8, 16, . . . 에서 뒤의 숫자는 앞의 숫자에 2를 곱해서 구할 수 있는 숫자이다. 일반적으로 우리는 Geometric Sequence는 아래처럼 일반화시켜서 적는다. $$ \{ a, ar, ar^2, ar^3, ar^4, . . . \} $$ * a 는 첫번째 숫자(term)을 말한다. * r 은 각 term의 (숫자) 요인이다 (공통요인, common ratio) 위에서 * a = 1 (첫번째 숫자) * r = 2 (두 숫자 간의 common ratio) \begin{eqnarray*} \{a, ar, ar^2, ar^3, . . . \} & = & \{1, 1 \text{x} 2^1, 1 \text{x} 2^2, 1 \text{x} 2^3, . . . \} \\ & = & \{1, 2, 4, 8, . . . \} \\ \text{where r should not be 0.} \end{eqnarray*} 몇 번째 숫자는 어떤 가치를 가질까?는 아래와 같이 구한다. $$ X_{n} = ar^{(n-1)} $$ 예를 들면 아래의 sequence가 있다고 할때, 5, 20, 80, 320, . . . * a = 5 * r = 4 * 4번째 숫자는 $5 \cdot 4^{4-1} = 320 $ * 10번째 숫자는 $5 \cdot 4^{9} = 5 \cdot 262144 = 1310720 $ r 이 -1 에서 1 사이의 값을 가질 때는 geometric sequence는 점점 작아진다. 예를 들면 $ \{12, 6, 3, 1.5, 0.75, . . . \}$ 에서의 계산 방법은 $ X_{n} = 12 \cdot (\frac{1}{2})^{n-1}$ 이다. 위에서 n이 무한대로 간다고 가정하면 이 때의 $X_{n} = 0$ 이 될 것이다. 즉, \begin{eqnarray*} X_{n} & = & ar^{(n-1)} \\ & & \text{where } -1 < r < +1 \\ & & \text{ and } n -> \infty \\ r^{(n-1)} & = & 0 \\ \therefore \text{ } ar^{(n-1)} & = & 0 \\ \therefore \text{ } X_{n} & = & 0 \\ \end{eqnarray*} ====== Sums of Geometric Series ====== Sequence의 숫자를 모두 더한다는 것은 아래를 뜻한다. $$ \large a + ar + ar^2 + . . . + ar^{(n-1)} $$ 이를 일반화하면 아래와 같다. $$ \sum_{k=0}^{n-1}(ar^{k}) = a \left(\frac{1-r^{n}}{1-r}\right)$$ 예를 들면 $$ \{ 10, 20, 40, 80, 160, . . . \}$$ 에서 처음 4개의 숫자를 더한 가치를 구해본다고 하면, \begin{eqnarray*} \sum_{k=0}^{n-1}(ar^{k}) & = & a \left(\frac{1-r^{n}}{1-r}\right) \\ & = & \sum_{k=0}^{4-1}(10 \cdot 2^{k}) \\ & = & 10 \left(\frac{1-2^4}{1-2}\right) \\ & = & 150 \end{eqnarray*} ====== Proof ====== \begin{eqnarray*} \sum_{k=0}^{n-1}(ar^{k}) = S & = & a + ar + ar^2 + ar^3 + . . . + ar^{(n-2)} + ar^{(n-1)} \\ \end{eqnarray*} 위의 식에 각 r을 곱한다면 \begin{eqnarray} S & = & a + ar + ar^2 + ar^3 + . . . + ar^{(n-2)} + ar^{(n-1)} \\ S\cdot r & = & \qquad ar + ar^2 + ar^3 + . . . + ar^{(n-2)} + ar^{(n-1)} + ar^{(n)} \\ \end{eqnarray} 위에서 아래를 빼면 \begin{eqnarray*} S-S\cdot r & = & a - ar^{n} \\ (1-r) \cdot S & = & a - ar^{n} \\ S & = & \frac {a - ar^{n}}{1-r} \\ & = & \frac {a (1 - r^{n})}{1-r} \\ & = & a \cdot \frac {(1 - r^{n})}{1-r} \\ \end{eqnarray*} ====== with Infinite Series (n이 무한대일 때) ====== n이 무한히 간다고 하고, r 이 -1 과 1 사이의 숫자라고 할 때 (여기서 -1, 0, 1은 포함하지 않는다. diminishing geometric series): \begin{eqnarray*} \sum_{n=0}^{\infty}(ar^n) & = & a \cdot \frac {(1 - r^{n})}{1-r} \\ & & \text{when } \\ & & n -> \infty, |r| < 1, r \ne 0 \\ & & r^{n} = 0 \\ \therefore \; \; \sum_{n=0}^{\infty}(ar^n) & = & a \cdot \left(\frac{1}{1-r}\right) \end{eqnarray*}